LSAT and Law School Admissions Forum

Get expert LSAT preparation and law school admissions advice from PowerScore Test Preparation.

 Administrator
PowerScore Staff
  • PowerScore Staff
  • Posts: 8916
  • Joined: Feb 02, 2011
|
#26249
Complete Question Explanation
(The complete setup for this game can be found here: lsat/viewtopic.php?t=10911)

The correct answer choice is (B)

If H is planted on lot 2, we need to conform to the 1-3-3 distribution in Template A, whereby H, O, and one other tree is planted on lot 2. As we already know, in that Template M must be planted on lot 3, because there can only be one tree in lot 1, and M and O cannot both be planted on lot 2. This prephrase that immediately proves answer choice (B) to be correct:
June15_game_2_#8_diagram_1.png
Answer choice (A): This answer choice is incorrect, because L can be planted on any of the lots.

Answer choice (B): This is the correct answer choice.

Answer choice (C): This answer choice is incorrect, because P can be planted on either lot 2 or 3.

Answer choice (D): This answer choice is incorrect, because S can be planted on either lot 2 or 3.

Answer choice (E): This answer choice is incorrect, because W cannot be planted on the same lot as M.
You do not have the required permissions to view the files attached to this post.

Get the most out of your LSAT Prep Plus subscription.

Analyze and track your performance with our Testing and Analytics Package.